Inscription / Connexion Nouveau Sujet
Niveau Maths sup
Partager :

suite

Posté par
cfg977
21-02-21 à 21:37

Bonsoir !
Je voudrais de l'aide pour faire l'exercice suivant :

Soit (U_n) une suite telle que pour tout n entier naturel, U_{n+2} <= \frac{U_{n+1}+U_n}{2}. Montrer que (U_n) converge.
J'ai considéré les suites (Vn) et (Wn) suivantes : Vn=max(Un,Un+1)
Wn=min(Un,Un+1).
Ma stratégie consiste à montrer que (Vn) et (Wn) sont convergentes. J'ai réussi à le faire pour (Vn) mais je n'y arrive pas  pour (Wn)

Posté par
verdurin
re : suite 21-02-21 à 21:46

Bonsoir,
la suite définie par un=-n vérifie la propriété demandé et n'est pas convergente.
Je crois que tu nous as caché une partie des hypothèses.

Posté par
cfg977
re : suite 21-02-21 à 21:53

En effet, (Un) est minorée.

Posté par
Ulmiere
re : suite 22-02-21 à 12:08

Ton inégalité est stable  par ajout d'une constante. Ca veut dire que (u_n+c) vérifie la même inégalité, pour tout c. Si tu la supposes minorée, tu peux donc sans perte de généralité la supposer positive. Mais nous on va faire mieux, on va travailler avec v_n = u_n -\dfrac{2u_1+u_0}{3}, de sorte que v_{n+2}\leqslant (v_{n+1}+v_n)/2 et v_1+v_0/2 = 0.

Pour tous N,n, v_{n+2}-v_{n+1} \leqslant -(v_{n+1}-v_n)/2. On somme entre n=0 et n = N-2.
Ca donne v_N \leqslant v_0/2 + v_1 -v_{N-1}/2 \leqslant -v_{N-1}/2
Donc v_N \leqslant v_{N-2}/4.

Par récurrence immédiate, on obient v_{2n} \leqslant v_0/2^{n+1} et v_{2n+1}\leqslant v_1/2^{n+1}. Ca veut dire que (v_n), et donc (u_n) et toutes ses translations, sont bornées (majorées par 1 par exemple, ou par n'importe quelle constante strictement positive si tu préfères). Tu peux donc appliquer Bolzano-Weierstrass à u, ou à n'importe quelle fonction continue de u. Je te laisse finir

Posté par
etniopal
re : suite 22-02-21 à 18:40

   Soit  a , b des réels > 0 et S(a,b) l'ensemble des suites u  : +  telles que u(0) = a , u(1) = b et  2u(n+2) u(n + 1) + u(n) pour tout n .
    S(a,b) contient la suite w qui vérifie  w(0) = a , w(1) = b et  2w(n+2)= w(n + 1) + w(n) pour tout n . On peut calculer explicitement les w(n)  et montrer que w converge  (vers (a + 2b)/3  si je ne me suis pas trompé).

   Soit maintenant  u  S(a,b)  .Il est clair que  u w  .
Bolzano-Weierstrass  permet d'affirmer que    u  , étant bornée ,  a  au moins une valeur d'adhérence .
Si on démontre qu'il n'y en a qu'une  on pourra dire qu'elle converge .

Posté par
jandri Correcteur
re : suite 23-02-21 à 08:45

Bonjour,

@Ulmiere : De v_N  \leqslant -v_{N-1}/2 on ne peut pas déduire v_N \leqslant v_{N-2}/4

Posté par
Ulmiere
re : suite 23-02-21 à 12:44

Effectivement, le signe moins inverse l'inégalité, cependant, on peut faire ceci :

Si tu refais la même chose avec N-1 à la place de N  : tu trouves v_{N-1} \leqslant -V_{N-2}/2 donc v_{N-2}/4 \leqslant v_N \leqslant -v_{N-1}/2.
Soit v est strictement négative, donc majorée.
Soit v prend une valeur positive, disons en N, et alors elle est encore positive en N+2, ce qui fournit une sous-suite positive.
Mais aussi, 0\geqslant -2v_{N+2} \geqslant v_{N+1}, et donc v_{N+1} est négative. Par récurrence, on a donc deux sous-suites, une positive et une négative. La négative est une suite bornée, donc on peut extraire une sous-suite convergente. Donc on a bien l'existence d'une sous-suite convergente de u (i.e d'une valeur d'adhérence).
Il reste à montrer que c'est la seule

Posté par
carpediem
re : suite 23-02-21 à 14:32

salut

une autre idée peut-être ...

u_{n + 2} \le\dfrac 1 2 (u_{n + 1}+ u_n) \iff 2u_{n + 2} \le u_{n + 1} + u_n

2u_2 \le u_1 + u_0
 \\ 2u_3 \le u_2 + u_1
 \\ 2u_4 \le u_3 + u_2
 \\ ...
 \\ 2u_{n - 2} \le u_{n - 3} + u_{n - 4}
 \\ 2u_{n - 1} \le u_{n - 2} + u_{n - 3}
 \\ 2u_n \le u_{n - 1} + u_{n - 2}

donc 2u_n + u_{n - 1} \le u_0 + 2u_1

or la suite (u_n) est minorée par un réel m donc pour tout entier n > 0  : 3m \le 2u_n + u_{n - 1} \le u_0 + 2u_1

si'il existait un indice n tel que u_n > \dfrac m 2 + u_0 + 2u_1 alors on en déduirait que  que u_{n - 1} < m

ce qui est contradictoire avec le fait que la suite est minorée par m

ce qui permet de conclure que la suite (u_n) est bornée (appartient même à l'intervalle [m, m/2 + u_0 + 2u_1[)

on peut donc en extraire une sous-suite convergente ...

Posté par
Ulmiere
re : suite 23-02-21 à 15:45

C'est joli. Tu utilises sans le dire une translation de u, par contre
Parce que u_{n-1}\leqslant u_0+2u_1 - 2u_n < u_0+2u_1-m-2(u_0+2u_1) \leqslant -m -(u_0+2u_1) n'est pas forcément < m a priori ?
Ou alors tu utilises peut-êter une autre égalité ?

En tout cas le même raisonnement fonctionne sans translation pour dire que u_n> -m/2+(u_0+2u_1)/2 n'est pas possible sinon u_{n-1} < u_0+2u_1+m-(u_0+2u_1)\leqslant m

Posté par
etniopal
re : suite 23-02-21 à 17:10

    Mais la question est   de  montrer que  toute u qui...  converge.

Posté par
Ulmiere
re : suite 23-02-21 à 17:17

Oui, mais on attend que l'auteur de la question fasse la suite

Posté par
carpediem
re : suite 23-02-21 à 18:51

ha oui !!! merci Ulmiere

je me suis mélangé un peu les pinceaux avec les inégalités en voulant tout faire de tête !!

pour la translation : oui ... mais en ai-je besoin avec mon raisonnement ?  sommer les inégalités comme je l'ai fait permet de s'en passer ... élégamment (en toute humilité !!)

de même (il me semble) faire une translation avec ton raisonnement n'est pas nécessaire mais fortement utile pour simplifier les calculs : sinon on se traine des constantes et à nouveau on tombe dans le pb de bien vérifier les signes et ordres des expressions manipulées ...

enfin ce me semble-t-il ...



etniopal @ 23-02-2021 à 17:10

    Mais la question est   de  montrer que  toute u qui...  converge.

en tout cas avec la première remarque de Ulmiere on peut considérer à translation près que la suite (u_n) est positive et bornée et que donc on peut en extraire une sous-suite convergente ...
Ulmiere @ 23-02-2021 à 17:17

Oui, mais on attend que l'auteur de la question fasse la suite
et j'avoue que je ne vois pas comment conclure pour l'instant ...

Posté par
Ulmiere
re : suite 23-02-21 à 18:53

Allez je balance, pour archive

0\leqslant v_{N+2n}\leqslant \dfrac{v_{N+2n-1}+v_{N+2(n-1)}}{2} \leqslant \dfrac{v_{N+2(n-1)}}{2} \leqslant \cdots \leqslant \dfrac{v_N}{2^n} \xrightarrow[n\to\infty]{}0

et

v_{N+1 + 2n} \geqslant v_{N+1+2(n-1)}/4 \geqslant \dots \geqslant v_{N+1}/4^n\xrightarrow[n\to\infty]{}0

donc 0\geqslant \limsup v_{N+1+2n} \geqslant \liminf v_{N+1+2n} \geqslant 0

D'où la convergence de (v_{N+n}) vers 0 donc de (v_n) vers 0, donc de (u_n) vers (2u_1+u_0)/3


Reste quand même à gérer le cas où v est à valeurs négatives, mais dans ce cas on refait comme pour v_{N+1} et on en déduit 0\geqslant v_{2n}\geqslant v_0/4^n \to 0
et 0\geqslant v_{2n+1} \geqslant 2v_{2(n+1)}-v_{2n}\to 2\times 0 - 0 = 0

Posté par
carpediem
re : suite 23-02-21 à 19:16

ha ouais !!

donc en fait l'existence d'une sous-suite convergente n'est d'aucune utilité !

en tout cas merci

Posté par
jandri Correcteur
re : suite 23-02-21 à 22:16


@Ulmiere ( message du 23-02-21 à 12:44) :

de v_N  \leqslant -\dfrac12v_{N-1} et de  v_{N-1} \leqslant -\dfrac12v_{N-2} tu ne peux pas déduire \dfrac14v_{N-2} \leqslant v_N \leqslant -\dfrac12v_{N-1}.

Ensuite, dans ton message du 23-02-21 à 18:53 tu écris :

0\leqslant v_{N+2n}\leqslant \dfrac{v_{N+2n-1}+v_{N+2(n-1)}}{2} \leqslant \dfrac{v_{N+2(n-1)}}{2}
donc tu supposes  v_{N+2n}\geqslant0 et  v_{N+2n-1}\leqslant0 alors qu'on ne connais pas le signe de v_n.

Enfin  (u_n) ne converge pas vers (2u_1+u_0)/3 en général, par exemple u_n=\dfrac1{2^n}

Posté par
jandri Correcteur
re : suite 23-02-21 à 22:28


Je propose une solution "élémentaire" au problème posé (convergence de la suite (u_n)).

On introduit la suite t_n=u_{n+1}+\dfrac12u_n qui est décroissante et minorée, donc convergente vers \ell.

En posant t_n=\ell+t'_n et u_n=\dfrac23\ell+u'_n on a t'_n=u'_{n+1}+\dfrac12u'_n.
On démontre que \lim (u'_n)=0 en utilisant \lim (t'_n)=0.

On a |t'_n|\leq\varepsilon pour n\geq n_0.
On montre par récurrence que u'_n=t'_{n-1}-\frac12t'_{n-2}+\dots +(-\frac12)^{n-n_0-1}t'_{n_0}+(-\frac12)^{n-n_0}u'_{n_0}

d'où |u'_n|\leq (1+\frac12+\dots +(\frac12)^{n-n_0-1})\,\varepsilon+\dfrac{|u'_{n_0}|}{2^{n-n_0}}\leq2\,\varepsilon+\dfrac{|u'_{n_0}|}{2^{n-n_0}}\leq3\,\varepsilon pour n\geq n_1

De manière analogue on peut démontrer que dans le cas où la suite (u_n) n'est pas minorée elle tend vers -\infty.

Posté par
DOMOREA
suite 24-02-21 à 11:25

bonjour,
Jandri , je ne comprends pas à quoi tu aboutis .

on démontre que t_n tend vers \frac{2u_1+u_0}{3} =l si tu veux donc t'_n tend vers 0,ok
à partir de 3ème ligne on sait que u'_n tend vers 0 mais en quoi cela te donne une information sur u_n?

Posté par
DOMOREA
suite 24-02-21 à 11:43

re,
je pensais plutôt suivre la même piste qu'etniopal.

Théorème de Bolzano Weierstrass: il existe au moins un point d'accumulation, Il s'agit de démontrer qu'il est unique.

J'ai trouvé une démonstration dont je suis à peu près sûr mais qui utilise le postulat qu'il existe un nombre fini p de points  A_1,A_2,...A_p d'accumulation. mais je suis certain que ce postulat est faux en général. Peut-être est-il possible que l'on puisse le poser dans le cas de cette suite.

Ensuite j'ai dit que l'ensemble des points d'accumulations est dénombrable, ceci est vrai car l'ensemble des termes de la suite est par définition dénombrable.
mais avec cette hypothèse je bloque sur une difficulté

Posté par
DOMOREA
suite 24-02-21 à 16:11

bonjour, ci-dessous un développement rapide sans couper les "e en 4"
qui est ma proposition pour démontrer que (u_n) converge en établissant qu'il ne peut pas y avoir plus d'un point d'accumulation.

Il y a sûrement à reprendre et à contester, je vous écoute si cela vous intéresse

je reprends en considérant qu'il existe une suite dénombrable de points d'accumulation A_k k\in\mathbb{N}pour la suite u_n.
Il est possible que inf(u_n) soit un point d'accumulation,  sinon il existe un  inf({A_k ,k \in \mathbb{N}})) on les notera A_1
A_1 et son suivant A_j étant séparés, il existe e_1>0 tel que ]A_1-e_1,A_1+e_1[\cap]A_j-e_j,A_J+e_j[=\emptyset on peut choisir e_1 tel que e_1<\frac{A_j-A_1}{3}
On notera désormais I_k=]a_k-e_k,A_k+e_k[

Soit u_n\in ]A_1-e_1,A_1+e_1[ il existe p\in \mathbb{N} tel que u_{n+p}\in ]A_1-e_1,A_1+e_1[ et u_{n+p+1}\notin ]A_1-e_1,A_1+e_1[
sinon A_1 serait le seul point d'accumulation ce qui termine la démonstration.
prenons l'hypothèse précédente  alors il existe k tel que u_{n+p+1} \in [A_1+e_1;max(u_0,u_1)] c'est à dire  entre un I_k  et son suivant ou à l'intérieur d'un I_k et k\ne 1

on a u_{n+p+2}\leq \frac{u_{n+p+1}+u_{n+p}}{2}}\leq\frac{au_{n+1}+bu_{n}}{a+b}a+b=2^{p-1}

ainsi \forall n'>n+p+2   on a u_{n'}\leq \frac{au_{n+1}+bu_{n}}{a+b} ce qui élimine les points A_j}  situés après u_{n+1}en tant que d'accumulation.

on réitère le raisonnement pour n'avoir plus qu'un point d'accumulation car au delà du dernier point d'accumulation , il n'y a qu'un nombre fini d'éléments de la suite, il suffit de travailler avec des n assez grand

Posté par
jandri Correcteur
re : suite 24-02-21 à 21:54

@ DOMOREA

la démonstration que j'ai proposée est élémentaire au sens où elle n'utilise que la définition d'une limite de suite (et pas d'autres notions de topologie comme les points d'accumulation).

Tu n'as peut-être pas lu attentivement ce que j'ai écrit. J'ai introduit t_n=u_{n+1}+\dfrac12u_n qui est une suite décroissante et minorée, donc convergente vers un réel \ell.

En posant t_n=\ell+t'_n et u_n=\dfrac23\ell+u'_n les deux suites t'_n et u'_n sont reliées par t'_n=u'_{n+1}+\dfrac12u'_n.
Comme \lim(t_n)=\ell on a \lim(t'_n)=0.
La partie la plus difficile est d'en déduire que \lim(u'_n)=0, ce qui entraine immédiatement que \lim(u_n)=\dfrac23\ell.
C'est pour montrer \lim(u'_n)=0 qu'on a besoin de la définition de la limite d'une suite (avec un \varepsilon).

Remarque : la limite de t_n n'est pas égale à \dfrac{2u_1+u_0}{3} en général, j'ai donné l'exemple de la suite u_n=\dfrac1{2^n}.

Posté par
jandri Correcteur
re : suite 24-02-21 à 22:12

Je propose une autre démonstration élémentaire utilisant la suite v_n=\max(u_n,u_{n+1}).
On montre d'abord que (v_n) est décroissante et minorée, donc convergente vers un réel \ell.

Pour tout \varepsilon>0 il existe donc n_0 tel que pour tout n\geq n_0 on ait \ell\leq v_n\leq \ell+\varepsilon.

Soit n>n_0. Il y a trois cas à distinguer.
Premier cas, u_n\geq u_{n+1} : u_n=v_n\in[\ell,\ell+\varepsilon].
Deuxième cas, u_n\geq u_{n-1} : u_n=v_{n-1}\in[\ell,\ell+\varepsilon].
Troisième cas, u_n< u_{n-1} et u_n< u_{n+1} : d'une part, u_n<v_n\leq\ell+\varepsilon
d'autre part, u_n\geq2u_{n+1}-u_{n-1}=2v_n-v_{n-1}\geq 2\ell-(\ell+\varepsilon)=\ell-\varepsilon.

Dans tous les cas on a \ell-\varepsilon\leq u_n\leq \ell+\varepsilon. Cela démontre que \lim(u_n)=\ell.

Posté par
DOMOREA
suite 25-02-21 à 10:37

bonjour jandri,
c'est en effet ta définition de u'_n que je n'avais pas saisie.
en fait on a tourné autour de la même idée avec ton t_n=\frac{2u_{n+1}+u_n}{2}
alors que moi je travaillais avec t_n=\frac{2u_{n+1}+u_n}{3} et si j'avais travaillé comme toi  mon u'_n aurait été défini par u_n=l+u'_n

Posté par
Sylvieg Moderateur
re : suite 25-02-21 à 19:23

Bonjour,
Bravo jandri pour tes démonstrations !
Très fort de trouver ces suites auxiliaires.
Et celle avec max ne coupe pas les en 4 \;

Posté par
jandri Correcteur
re : suite 25-02-21 à 21:15

Bonsoir Sylvieg,

merci pour tes compliments mais j'avais déjà cherché cet exercice et j'ai pris des idées chez d'autres.

L'exercice peut se généraliser ainsi : soit a\in]-1,1[ et (u_n) une suite vérifiant pour tout n
u_{n+2}\leq (1-a)u_{n+1}+au_n . Montrer que la suite (u_n) converge ou bien diverge vers -\infty.

La première démonstration que j'ai proposée s'applique sans problème mais la seconde (avec le max) ne s'applique que pour a\in[0,1[ .

Si on renverse l'inégalité dans la définition de (u_n) on obtient que (u_n) converge ou bien diverge vers +\infty.

Posté par
Sylvieg Moderateur
re : suite 26-02-21 à 09:15

Le silence de cfg977 est assourdissant.
Pourtant, il avait introduit une des suites qui permet d'aboutir dès son 1er message :

Citation :
Vn=max(Un,Un+1)

PS Si on renverse l'inégalité, il suffit de poser rn = -un ?

Posté par
jandri Correcteur
re : suite 26-02-21 à 10:49

Oui Sylvieg, c'est exactement cela.



Vous devez être membre accéder à ce service...

Pas encore inscrit ?

1 compte par personne, multi-compte interdit !

Ou identifiez-vous :


Rester sur la page

Inscription gratuite

Fiches en rapport

parmi 1674 fiches de maths

Désolé, votre version d'Internet Explorer est plus que périmée ! Merci de le mettre à jour ou de télécharger Firefox ou Google Chrome pour utiliser le site. Votre ordinateur vous remerciera !